Ανισότητα

Συντονιστές: cretanman, Demetres, polysot, socrates, silouan

socrates
Επιμελητής
Δημοσιεύσεις: 6461
Εγγραφή: Δευ Μαρ 09, 2009 1:47 pm
Τοποθεσία: Θεσσαλονίκη
Επικοινωνία:

Ανισότητα

#1

Μη αναγνωσμένη δημοσίευση από socrates » Τετ Νοέμ 11, 2015 12:05 am

Έστω a, b, c θετικοί πραγματικοί αριθμοί τέτοιοι ώστε a^3 + b^3 + c^3 = a^4 + b^4 + c^4. Να δείξετε ότι

\displaystyle{\frac{a}{a^2 + b^4 + c^4}+\frac{b}{a^4 + b^2 + c^4}+\frac{c}{a^4 + b^4 + c^2}\geq  1.}


Θανάσης Κοντογεώργης
Άβαταρ μέλους
Ορέστης Λιγνός
Δημοσιεύσεις: 1835
Εγγραφή: Κυρ Μάιος 08, 2016 7:19 pm
Τοποθεσία: Χαλάνδρι Αττικής
Επικοινωνία:

Re: Ανισότητα

#2

Μη αναγνωσμένη δημοσίευση από Ορέστης Λιγνός » Κυρ Ιουν 19, 2016 6:30 pm

socrates έγραψε:Έστω a, b, c θετικοί πραγματικοί αριθμοί τέτοιοι ώστε a^3 + b^3 + c^3 = a^4 + b^4 + c^4. Να δείξετε ότι

\displaystyle{\frac{a}{a^2 + b^4 + c^4}+\frac{b}{a^4 + b^2 + c^4}+\frac{c}{a^4 + b^4 + c^2}\geq  1.}
Επαναφορά!


Κερδίζουμε ό,τι τολμούμε!
Άβαταρ μέλους
ΦΩΤΙΑΔΗΣ ΠΡΟΔΡΟΜΟΣ
Δημοσιεύσεις: 921
Εγγραφή: Πέμ Νοέμ 22, 2018 9:43 pm

Re: Ανισότητα

#3

Μη αναγνωσμένη δημοσίευση από ΦΩΤΙΑΔΗΣ ΠΡΟΔΡΟΜΟΣ » Κυρ Απρ 12, 2020 7:06 pm

socrates έγραψε:
Τετ Νοέμ 11, 2015 12:05 am
Έστω a, b, c θετικοί πραγματικοί αριθμοί τέτοιοι ώστε a^3 + b^3 + c^3 = a^4 + b^4 + c^4. Να δείξετε ότι

\displaystyle{\frac{a}{a^2 + b^4 + c^4}+\frac{b}{a^4 + b^2 + c^4}+\frac{c}{a^4 + b^4 + c^2}\geq  1.}
Πολύ καλό :P
Από \rm B-C-S είναι
\displaystyle {\rm \left ( \sum a^2 \right )\left ( \sum a^3 \right )=\left (\sum a^2  \right )\left ( \sum a^4 \right )\geq \left ( \sum a^3 \right )^2\Leftrightarrow \sum a^2\geq \sum a^3}.Άρα
\displaystyle{\rm \sum a^3\sum a^2\leq \left ( \sum a^2 \right )^2\overset{B-C-S}{\leq} \sum a\sum a^3\Leftrightarrow \sum a\geq \sum a^2}.
Τα από κάτω δεν χρειάζονται..δείτε την συνέχεια σε επόμενο ποστ
Είναι
\displaystyle {\rm \sum a\geq \sum a^2\Leftrightarrow 3\sum a\geq \sum a^2+2\sum a^4\Leftrightarrow 9\sum a\geq 3\left ( \sum a^2+2\sum a^4 \right )}
\displaystyle {\rm \Leftrightarrow \dfrac{3}{\displaystyle\sum a}\leq \dfrac{9}{\displaystyle\sum a^2+\sum (b^4+c^4)}\overset{(*)}{\leq }\sum \dfrac{1}{a^2+b^4+c^4}\Leftrightarrow \sum \dfrac{1}{a^2+b^4+c^4}\sum a\geq 3}.
Η (*) προκύπτει από την \displaystyle{\rm \sum a\sum \dfrac{1}{a}\geq 9}.
Επειδή οι \rm (a,b,c),(\dfrac{1}{a^2+b^4+c^4},\dfrac{1}{b^2+a^4+c^4},\dfrac{1}{c^2+b^4+a^4}) είναι όμοια διατεταγμένες από \rm Chebicheff θα έχουμε
\displaystyle{\rm 3\leq \sum a\sum \dfrac{1}{a^2+b^4+c^4}\leq 3\sum \dfrac{a}{a^2+b^4+c^4}\Leftrightarrow \sum \dfrac{a}{a^2+b^4+c^4}\geq 1} και η απόδειξη ολοκληρώθηκε.
τελευταία επεξεργασία από ΦΩΤΙΑΔΗΣ ΠΡΟΔΡΟΜΟΣ σε Τρί Απρ 14, 2020 1:05 pm, έχει επεξεργασθεί 1 φορά συνολικά.


socrates
Επιμελητής
Δημοσιεύσεις: 6461
Εγγραφή: Δευ Μαρ 09, 2009 1:47 pm
Τοποθεσία: Θεσσαλονίκη
Επικοινωνία:

Re: Ανισότητα

#4

Μη αναγνωσμένη δημοσίευση από socrates » Τρί Απρ 14, 2020 12:03 am

ΦΩΤΙΑΔΗΣ ΠΡΟΔΡΟΜΟΣ έγραψε:
Κυρ Απρ 12, 2020 7:06 pm
socrates έγραψε:
Τετ Νοέμ 11, 2015 12:05 am
Έστω a, b, c θετικοί πραγματικοί αριθμοί τέτοιοι ώστε a^3 + b^3 + c^3 = a^4 + b^4 + c^4. Να δείξετε ότι

\displaystyle{\frac{a}{a^2 + b^4 + c^4}+\frac{b}{a^4 + b^2 + c^4}+\frac{c}{a^4 + b^4 + c^2}\geq  1.}
...

Επειδή οι \rm (a,b,c),(\dfrac{1}{a^2+b^4+c^4},\dfrac{1}{b^2+a^4+c^4},\dfrac{1}{c^2+b^4+a^4}) είναι όμοια διατεταγμένες

Δε βλέπω γιατί ισχύει αυτό... :?


Θανάσης Κοντογεώργης
Άβαταρ μέλους
ΦΩΤΙΑΔΗΣ ΠΡΟΔΡΟΜΟΣ
Δημοσιεύσεις: 921
Εγγραφή: Πέμ Νοέμ 22, 2018 9:43 pm

Re: Ανισότητα

#5

Μη αναγνωσμένη δημοσίευση από ΦΩΤΙΑΔΗΣ ΠΡΟΔΡΟΜΟΣ » Τρί Απρ 14, 2020 1:03 pm

socrates έγραψε:
Τρί Απρ 14, 2020 12:03 am
ΦΩΤΙΑΔΗΣ ΠΡΟΔΡΟΜΟΣ έγραψε:
Κυρ Απρ 12, 2020 7:06 pm
socrates έγραψε:
Τετ Νοέμ 11, 2015 12:05 am
Έστω a, b, c θετικοί πραγματικοί αριθμοί τέτοιοι ώστε a^3 + b^3 + c^3 = a^4 + b^4 + c^4. Να δείξετε ότι

\displaystyle{\frac{a}{a^2 + b^4 + c^4}+\frac{b}{a^4 + b^2 + c^4}+\frac{c}{a^4 + b^4 + c^2}\geq  1.}
...

Επειδή οι \rm (a,b,c),(\dfrac{1}{a^2+b^4+c^4},\dfrac{1}{b^2+a^4+c^4},\dfrac{1}{c^2+b^4+a^4}) είναι όμοια διατεταγμένες

Δε βλέπω γιατί ισχύει αυτό... :?
Πράγματι δεν ισχύει γενικά :oops:

Συνεχίσω από εκεί που έδειξα πως \displaystyle{\rm \sum a\geq \sum a^2} ,τα από κάτω μετά δεν χρειάζονται.
Η αρχική γράφεται \displaystyle {\rm \sum \dfrac{a^2}{a^3+a\left ( b^4+c^4 \right )}\geq 1}.
Από \rm Andreesqu αρκεί \rm \displaystyle {\rm \left ( \sum a \right )^2\geq \sum a^3+\sum a(b^4+c^4)=\sum a^3+\sum a\left ( \sum a^3-a^4 \right )=}
\displaystyle {\rm =\sum a^3+\sum a\sum a^3-\sum a^5}.
Από \rm B-C-S είναι \displaystyle {\rm \sum a^3\sum a^5\geq \left ( \sum a^3 \right )^2\Leftrightarrow \sum a^5\geq \sum a^3 }.
Άρα αρκεί  \displaystyle {\rm \left (\sum a   \right )^2\geq \sum a\sum a^3\Leftrightarrow \sum a\geq \sum a^3} το οποίο ισχύει αφού \displaystyle {\rm \sum a\geq \sum a^2\geq \sum a^3}

Ελπίζω τώρα να είναι εντάξει :?


socrates
Επιμελητής
Δημοσιεύσεις: 6461
Εγγραφή: Δευ Μαρ 09, 2009 1:47 pm
Τοποθεσία: Θεσσαλονίκη
Επικοινωνία:

Re: Ανισότητα

#6

Μη αναγνωσμένη δημοσίευση από socrates » Τετ Απρ 15, 2020 7:43 pm

ΦΩΤΙΑΔΗΣ ΠΡΟΔΡΟΜΟΣ έγραψε:
Τρί Απρ 14, 2020 1:03 pm
socrates έγραψε:
Τρί Απρ 14, 2020 12:03 am
ΦΩΤΙΑΔΗΣ ΠΡΟΔΡΟΜΟΣ έγραψε:
Κυρ Απρ 12, 2020 7:06 pm
socrates έγραψε:
Τετ Νοέμ 11, 2015 12:05 am
Έστω a, b, c θετικοί πραγματικοί αριθμοί τέτοιοι ώστε a^3 + b^3 + c^3 = a^4 + b^4 + c^4. Να δείξετε ότι

\displaystyle{\frac{a}{a^2 + b^4 + c^4}+\frac{b}{a^4 + b^2 + c^4}+\frac{c}{a^4 + b^4 + c^2}\geq  1.}
...

Επειδή οι \rm (a,b,c),(\dfrac{1}{a^2+b^4+c^4},\dfrac{1}{b^2+a^4+c^4},\dfrac{1}{c^2+b^4+a^4}) είναι όμοια διατεταγμένες

Δε βλέπω γιατί ισχύει αυτό... :?
Πράγματι δεν ισχύει γενικά :oops:

Συνεχίσω από εκεί που έδειξα πως \displaystyle{\rm \sum a\geq \sum a^2} ,τα από κάτω μετά δεν χρειάζονται.
Η αρχική γράφεται \displaystyle {\rm \sum \dfrac{a^2}{a^3+a\left ( b^4+c^4 \right )}\geq 1}.
Από \rm Andreesqu αρκεί \rm \displaystyle {\rm \left ( \sum a \right )^2\geq \sum a^3+\sum a(b^4+c^4)=\sum a^3+\sum a\left ( \sum a^3-a^4 \right )=}
\displaystyle {\rm =\sum a^3+\sum a\sum a^3-\sum a^5}.
Από \rm B-C-S είναι \displaystyle {\rm \sum a^3\sum a^5\geq \left ( \sum a^3 \right )^2\Leftrightarrow \sum a^5\geq \sum a^3 }.
Άρα αρκεί  \displaystyle {\rm \left (\sum a   \right )^2\geq \sum a\sum a^3\Leftrightarrow \sum a\geq \sum a^3} το οποίο ισχύει αφού \displaystyle {\rm \sum a\geq \sum a^2\geq \sum a^3}

Ελπίζω τώρα να είναι εντάξει :?

Ωραία! :coolspeak:


Θανάσης Κοντογεώργης
Απάντηση

Επιστροφή σε “Άλγεβρα - Θεωρία Αριθμών - Συνδυαστική (Juniors) - Παλαιότερες Συζητήσεις”

Μέλη σε σύνδεση

Μέλη σε αυτήν τη Δ. Συζήτηση: Δεν υπάρχουν εγγεγραμμένα μέλη και 1 επισκέπτης